Đến nội dung

royal1534 nội dung

Có 759 mục bởi royal1534 (Tìm giới hạn từ 03-05-2020)



Sắp theo                Sắp xếp  

#651365 BẤT ĐẲNG THỨC QUA CÁC KÌ THI HỌC SINH GIỎI LỚP 12 CẤP THÀNH PHỐ

Đã gửi bởi royal1534 on 26-08-2016 - 18:06 trong Bất đẳng thức và cực trị

Bài 15: Cho $x,y,z$ là các số thực không âm thỏa mãn: $xy+yz+zx=1$. Tìm GTNN của biểu thức:

$P=\frac{1}{x^2+y^2}+\frac{1}{y^2+z^2}+\frac{1}{z^2+x^2}+\frac{5}{2}(x+1)(y+1)(z+1)$.

 

Ta có bổ đề sau:

Với $x,y,z>0. Thì \sum \frac{1}{x^2+y^2} \geq \frac{10}{(x+y+z)^2}$

-----

Áp dụng bổ đề ta có:

$VT \geq \frac{10}{(x+y+z)^2}+\frac{5}{2}(x+1)(y+1)(z+1)$ $=\frac{10}{(x+y+z)^2}+\frac{5}{2}(xyz+\sum x+\sum xy+1) $$\geq \frac{10}{(x+y+z)^2}+\frac{5}{2}(\sum x+2)$ (Vì $xyz \geq 0, xy+yz+zx=1$)

Đặt $x+y+z=p (p>0)$

Ta có $VT \geq \frac{10}{p^2}+\frac{5}{2}(p+2)=\frac{10}{p^2}+\frac{5}{4}p+\frac{5}{4}p+5 \geq 3\sqrt[3]{\frac{10.5.5.p^2}{4.4.p}}+5=\frac{5}{2}+5=\frac{25}{2}$

Dấu '=' xảy ra khi $x=y=1,z=0$

-------

P/s : 1, Chứng minh bổ đề trên có thể xem tại đây http://www.artofprob...1122010p5166529

       2, Em nghỉ để duy trì tính thẩm mĩ topic thì anh nên đăng l lần 2-3 bài ạ. :D




#651306 Topic: [LTDH] Mỗi ngày hai bất đẳng thức.

Đã gửi bởi royal1534 on 25-08-2016 - 23:53 trong Bất đẳng thức và cực trị

Bài 38: Cho $a,b,c>0$ thỏa mãn: $a+b+c=1$. Chứng minh rằng:

$\frac{ab+bc+ca}{a^2b^2+b^2c^2+c^2a^2}\ge 8(a^2+b^2+c^2)$.

Hình như đề phải là $a,b,c \geq 0$ anh nhỉ ...

Đổi biến $(a+b+c,abc,ab+bc+ca)=(p,r,q)$

Ta có đánh giá: $q^2 \geq 3pr=3r>2r \rightarrow q^2-2r>0$

Với $p=1$. BĐT cần chứng minh tương đương với:

$\frac{q}{q^2-2r} \geq 8(1-2q)$

$\Leftrightarrow 16r+\frac{q(4q-1)^2}{1-2q} \geq 0$ (Đúng vì $q \leq \frac{p^2}{3}=\frac{1}{3}<\frac{1}{2}$)

Ta có đpcm. Dấu '=' xảy ra khi $a=b=\frac{1}{2},c=0$




#650298 $\fbox{Đề thi học sinh giỏi toán 10 Trường PTTH chuyên Khoa họ...

Đã gửi bởi royal1534 on 19-08-2016 - 00:09 trong Thi HSG cấp Tỉnh, Thành phố. Olympic 30-4. Đề thi và kiểm tra đội tuyển các cấp.

 

Bài 3: Cho $x,y,z >0, x+y+z=xyz$. Chứng minh rằng

$\sqrt{x^2+y^2}+\sqrt{y^2+z^2}+\sqrt{z^2+x^2}+3\sqrt{6}\leq \sqrt{8}xyz$

 

Nhắm làm được mỗi bài này :D.

----------------

Lời giải:

Giả thiết đã cho $\Leftrightarrow \frac{1}{xy}+\frac{1}{yz}+\frac{1}{zx} \leq 1 $

Đổi biến $(\frac{1}{x},\frac{1}{y},\frac{1}{z})=(a,b,c) $

$\Rightarrow ab+bc+ca=1$

BĐT cần chứng minh được viết lại thành:

$c\sqrt{a^2+b^2}+b\sqrt{c^2+a^2}+a\sqrt{b^2+c^2}+3\sqrt{6}abc \leq \sqrt{8}$

Từ giả thiết ta có

$1 \geq 3\sqrt[3]{a^2b^2c^2}$

$\Leftrightarrow abc \leq \frac{1}{3\sqrt{3}}$

Sử dụng bđt trên ta có: $(c\sqrt{2ab}+b\sqrt{2ac}+a\sqrt{2bc}) \geq 3\sqrt[3]{2\sqrt{2}a^2b^2c^2} \geq 3\sqrt{6}abc$

Suy ra $VT \leq c(\sqrt{a^2+b^2}+\sqrt{2ab})+b(\sqrt{c^2+a^2}+\sqrt{2ca})+a(\sqrt{b^2+c^2}+\sqrt{2bc}) \leq \sum c\sqrt{2(a^2+b^2+2ab)}=\sum \sqrt{2} c(a+b)=2\sqrt{2}$

Ta có điều phải chứng minh. Dấu '=' xảy ra khi $x=y=z=\sqrt{3}$




#650250 CMR tồn tại vô hạn số tự nhiên N để $(4n^2+1)\vdots 5$ hoặc...

Đã gửi bởi royal1534 on 18-08-2016 - 18:32 trong Đại số

 

2. Tồn tại hay không số tự nhiên n mà $n^2 + n + 1 \vdots 1995$

 

 

Ta có $n^3-1=(n-1)(n^2+n+1) \vdots 1995$

$\Rightarrow n^3=1$ (mod $1995$)

$\Rightarrow n^3=1$ (mod $5$) $\rightarrow n^4 \equiv n$ (mod $5$)  (1)

-Xét $5 \mid n$. Lúc đó ta có $n^2+n+1$ không chia hết cho 5 (Loại)

-Xét $(5,n)=1$. Theo định lý Fermat nhỏ ta có: $n^4 \equiv 1 (mod 5)$ (2)

Từ (1)(2) $\Rightarrow n \equiv 1$ (mod 5)

Lúc đó ta lại có $n^2+n+1 \equiv 3$ (mod 5) (Loại)

Vậy không tồn tại số tự nhiên $n$ thỏa đề bài 




#650090 $2\sqrt{x}+\frac{32}{(2\sqrt...

Đã gửi bởi royal1534 on 17-08-2016 - 18:32 trong Phương trình - hệ phương trình - bất phương trình

Giải hệ phương trình:$\left\{\begin{matrix} 2\sqrt{x}+\frac{32}{(2\sqrt{y-3}+3)^2}=5\\\sqrt{\sqrt{x}(2\sqrt{x}+\sqrt{y-3}+1)}+\sqrt{(\sqrt{y-3}+1)(\sqrt{x}+2\sqrt{y-3}+2)}=\sqrt{6(x+(\sqrt{y-3}+1)^2)}  \end{matrix}\right.$

Đặt: $\sqrt{x}=a, \sqrt{y-3}+1=b$ ($a \geq 0, b \geq 1$)

 

$$PT(2) \Leftrightarrow \sqrt{2a^2+ab}+\sqrt{2b^2+ab}=6\sqrt{a^2+b^2}$$

 

Áp dụng bất đẳng thức AM-GM và bất đẳng thức Cauchy-Schwarz ta có:

 

$$VT \leq \sqrt{2(2a^2+ab)+2(b^2+ab)}=\sqrt{4(a^2+b^2)+4ab} \leq \sqrt{6(a^2+b^2)}=VP$$

 

Dấu $'='$ xảy ra khi $a=b$

 

Viết lại PT(1) ta có: $$2a+\frac{32}{(2b+1)^2}=5$$

 

$$\Leftrightarrow 2a+\frac{32}{(2a+1)^2}=5$$

 

$$\Leftrightarrow 2a(2a+1)^2+32-5(2a+1)^2=0$$

 

$$\Leftrightarrow (2a-3)^2(2a+3)=0$$

 

$$\Leftrightarrow a=\frac{3}{2} ( a \geq 0)$$

 

$$\Leftrightarrow \sqrt{x}=\frac{3}{2} \Leftrightarrow x=\frac{9}{4}$$

 

Ta có $$b=a=\frac{3}{2} \Leftrightarrow \sqrt{y-3}+1=\frac{3}{2} \Leftrightarrow y=\frac{13}{4}$$

 

Vậy $(x,y)=(\frac{9}{4},\frac{13}{4})$




#649869 Topic: [LTDH] Mỗi ngày hai bất đẳng thức.

Đã gửi bởi royal1534 on 16-08-2016 - 13:12 trong Bất đẳng thức và cực trị

Hai bài: Bài 17Bài 18 mọi người đã giải quá chuẩn rồi, nên mình đề xuất hai bài tiếp theo như sau:

Bài 19:Cho $x,y,z>0$. Chứng minh rằng:

$\frac{1}{\sqrt{x^2+y^2+z^2+1}}-\sqrt[3]{\frac{3}{x+y+z}(\frac{x^3}{xy+2yz}+\frac{y^3}{yz+2xz}+\frac{z^3}{xz+2xy})}-\frac{2}{(1+x)(1+y)(1+z)}\le \frac{-3}{4}$.

 

Chém nốt :P.

-------

Áp dụng bất đẳng thức Holder ta có:

 

$\sum \frac{x^3}{xy+2yz} \geq \frac{(x+y+z)^3}{3\sum (xy+2yz)}=\frac{(x+y+z)^3}{9(xy+yz+zx)}$

 

$\Rightarrow \frac{3}{x+y+z}.\sum \frac{x^3}{xy+2yz} \geq \frac{(x+y+z)^2}{3(xy+yz+zx)} \geq 1$

 

Suy ra: $-\sqrt[3]{\frac{3}{x+y+z}.\sum \frac{x^3}{xy+2yz}} \leq -1 $

 

Công việc còn lại ta chỉ cần chứng minh: 

 

$\frac{1}{\sqrt{x^2+y^2+z^2+1}}-\frac{2}{(1+x)(1+y)(1+z)} \leq \frac{1}{4}$

 

Áp dụng bđt Cauchy-Schwarz ta có: 

 

$\frac{1}{\sqrt{x^2+y^2+z^2+1}} \leq \frac{2}{x+y+z+1}$

 

Vậy ta cần chứng minh: $\frac{2}{x+y+z+1}-\frac{2}{(1+x)(1+y)(1+z)} \leq \frac{1}{4}$

 

Đặt $x+1=a, y+1=b, z+1=c$  Vì $x,y,z>0$ suy ra $abc >1$

 

Viết lại bđt cần chứng minh thành:

 

$\frac{2}{a+b+c-2}-\frac{2}{abc} \leq \frac{1}{4}$

 

$\leftrightarrow \frac{1}{a+b+c-2}-\frac{1}{abc} \leq \frac{1}{8}$

 

Quy đồng và biến đổi tương đương. BĐT trên tương đương với:

 

$10abc-8(a+b+c)-abc(a+b+c)+16 \leq 0 $

 

Đổi biến $(abc,a+b+c)=(r,p)$ $(r>1)$

 

Ta cần chứng minh $10r-8p-pr+16 \leq 0 \leftrightarrow 8p+pr-10r-16 \geq 0$

 

Chú ý theo bất đẳng thức AM-GM thì $p \geq 3\sqrt[3]{r}$

 

Tiếp tục đặt $\sqrt[3]{r}=t$ $(t>1)$

 

Thì ta cần chứng minh $24t+3t^4-10t^3-16 \geq 0$

 

$\leftrightarrow (t-2)^2(3t^2+2t-4) \geq 0$: Đúng vì $t > 1$

 

Chứng minh hoàn tất. Dấu '=' xảy ra khi $x=y=z=1$




#649830 $p^3-2p^2+p+1=3^n$

Đã gửi bởi royal1534 on 16-08-2016 - 00:26 trong Số học

Câu này trog đề trại hè Hùng Vương lớp 10 mới thi xg  :icon9:

Quào. Bạn có lời giải khác hay có link bài toán cho mình xin được không :)




#649770 Topic: [LTDH] Mỗi ngày hai bất đẳng thức.

Đã gửi bởi royal1534 on 15-08-2016 - 17:45 trong Bất đẳng thức và cực trị

Lời giải bài 15bài 16 giống hoàn toàn lời giải bài bạn nguyenhongsonk612tpdtthltvp nên mình đề xuất hai bài tiếp theo như sau:

Bài 17: Cho các số thực $x,y,z$ thay đổi thỏa mãn: $x^2+y^2+z^2=1$. Tìm GTNN của biểu thức:

$P=(xy+yz+2zx)^2-\frac{8}{(x+y+z)^2-xy-yz+2}$

Bài 18: Cho $x,y,z$ là ba số thực thỏa mãn: $2x+3y+z=40$. Tìm GTNN của biểu thức:

$S=2\sqrt{x^2+1}+3\sqrt{y^2+16}+\sqrt{z^2+36}$ 

Thấy mọi người giải nhanh quá nên không kịp giải :D

--------

Bài 17:

Sử dụng giả thiết $x^2+y^2+z^2=1$. Ta viết lại $P=(xy+yz+2zx)^2-\frac{8}{(x+y+z)^2-xy-yz+2}=(xy+yz+2zx)^2-\frac{8}{3+xy+yz+2zx}$

Bây giờ ta chỉ cần tìm Min của $xy+yz+2zx$ là bài toán được giải quyết

Hiển nhiên ta có:

-$(x+y+z)^2 \geq 0$

-$2zx \geq -(x^2+z^2)=-(1-y^2)=y^2-1$

$\Rightarrow 2xy+2yz+4zx+1 \geq y^2-1$

$\Leftrightarrow 2xy+2yz+4zx \geq y^2-2 \geq -2$

$\Leftrightarrow xy+yz+2zx \geq -1$

------

Dự đoán $MinP=(-3)$ ta đi chứng minh 

 $P=(xy+yz+2zx)^2-\frac{8}{3+xy+yz+2zx} \geq -3$

$\Leftrightarrow t^2-\frac{8}{3+t} \geq -3$ (Với $t =xy+yz+2zx \geq -1$)

$\Leftrightarrow (t+1)^3 \geq 0$: Đúng vì $t \geq -1$

Dấu '=' xảy ra khi bộ $(x,y,z)$ thõa: $x=-z,y=0,x^2+y^2+z^2=1$




#649768 $p^3-2p^2+p+1=3^n$

Đã gửi bởi royal1534 on 15-08-2016 - 17:25 trong Số học

Tìm số nguyên dương $n$ và số nguyên tố $p$ thỏa 

$p^3-2p^2+p+1=3^n$




#648603 Chứng minh $S_{k}$ đồng dư $0$ (mod $p...

Đã gửi bởi royal1534 on 08-08-2016 - 18:39 trong Số học

Giả sử $p$ là số nguyên tổ lẻ. $S_{k}$ là tổng các tích gồm $k$ số phân biệt lấy từ tập {$1,2,....,p-1$}

Chứng minh $S_{k}$ đồng dư $0$ (mod $p$) với $2 \leq k \leq p-2$




#648032 CMR $\frac{x}{x\sqrt{x+yz}}+...

Đã gửi bởi royal1534 on 05-08-2016 - 12:16 trong Bất đẳng thức và cực trị

Cho ba số thực dương x, y, z thỏa mãn x + y + z = 1
CMR $\frac{x}{x\sqrt{x+yz}}+\frac{y}{y+\sqrt{y+zx}}+\frac{z}{z+\sqrt{z+xy}}\leq 1$

Áp dụng bất đẳng thức Cauchy-Schwarz ta có:

$x+\sqrt{x+yz}=x+\sqrt{x(x+y+z)+yz}=x+\sqrt{(x+y)(x+z)} \geq x+\sqrt{xy}+\sqrt{xz}$

Suy ra $\frac{x}{x+\sqrt{x+yz}} \leq \frac{x}{x+\sqrt{xy}+\sqrt{zx}}=\frac{\sqrt{x}}{\sqrt{x}+\sqrt{y}+\sqrt{z}}$

Thiết lập 2 bđt tương tự và cộng lại ta có đpcm. Dấu '=' xảy ra khi $x=y=z=\frac{1}{3}$




#647902 Tìm số nguyên dương n sao cho n chia hết cho mọi số nguyên dương không vượt q...

Đã gửi bởi royal1534 on 04-08-2016 - 13:26 trong Số học

Tìm số nguyên dương n sao cho n chia hết cho mọi số nguyên dương không vượt quá $\sqrt{n}$




#647711 Chứng minh $\sum\frac{1}{(a+b+2\sqrt{...

Đã gửi bởi royal1534 on 02-08-2016 - 23:51 trong Bất đẳng thức và cực trị

Cho a,b,c dương thoả mãn 

$\frac{1}{a}+\frac{1}{b}+\frac{1}{c}\leq 16(a+b+c)$

Chứng minh

$\sum\frac{1}{(a+b+\sqrt{2(a+c)})^3}\leq\frac{8}{9}$

 

Hint

Áp dụng bất đẳng thức AM-GM ta có 

$(a+b+\frac{\sqrt{2(a+c)}}{2}+\frac{\sqrt{2(a+c)}}{2})^3 \geq \frac{27}{2(a+b)(a+c)}$

Suy ra $VT \leq \sum \frac{2}{27(a+b)(a+c)}$

Và ta quy bài toán về chứng minh $\sum \frac{1}{(a+b)(a+c)} \leq 12 $

$\Leftrightarrow (a+b+c) \leq 6(a+b)(b+c)(c+a)$

Giả thiết tương đương với $16(a+b+c)abc \geq ab+bc+ca$

Mà theo bđt AM-GM thì ta có: $(ab+bc+ca)^2 \geq 3(a+b+c)abc \geq \frac{3}{16}(ab+bc+ca)$

$\Leftrightarrow ab+bc+ca \geq \frac{3}{16}$

Áp dụng bổ đề quen thuộc trên (Chứng minh khá dễ). Ta có

$6(a+b)(b+c)(c+a) \geq 6.\frac{8}{9}.(a+b+c)(ab+bc+ca) \geq 6.\frac{8}{9}.\frac{3}{16}(a+b+c)=a+b+c $

Ta có Q.E.D. Dấu '=' xảy ra khi $a=b=c=\frac{1}{4}$




#647144 Cho x, y, z>0 : CMR: $\sum \frac{x^{3}...

Đã gửi bởi royal1534 on 30-07-2016 - 08:48 trong Bất đẳng thức và cực trị

À với cách đặt như vậy thì $abc=1$ nên em liên tưởng tới BĐT $vasc: \;\;\;\;\;\;\;\ \sum \frac{1}{a^{2k}+a^{k}+1}\geq 1.$

Ta sẽ chứng minh: $\frac{1}{(1+a)^3}\geq \frac{3}{8(a^{2k}+a^k+1)}\Leftrightarrow 8a^{2k}+8a^k+5\geq 3a^3+9a^2+9a$

Để có được như vậy thì đẳng thức phải xảy ra, tức là $8a^{2k}+8a^k+5= 3a^3+9a^2+9a$

Đạo hàm cả $2$ vế thì được $16k.a^{2k-1}+8k.a^{k-1}=9a^2+18a+9$

Mà dấu $"="$ xảy ra khi $x=y=z$ nên $a=1,$ từ đó tính được $k=\frac{3}{2}.$

Việc cuối cùng chỉ là đi chứng minh BĐT: $\frac{1}{(1+a)^3}\geq \frac{3}{8(a^3+\sqrt{a^3}+1)}$ là đúng.

---------------------------------------

Em cũng không biết tại sao anh lại nghĩ đến chứng minh BĐT: $(1+a)^3 \leq (1+abc)(1+\frac{a}{b})(1+\frac{a}{c})$

Chỉ là bài toán anh đọc qua rồi. Nhưng anh nghĩ mò mẩm để cho ra bđt quen thuộc là chính :~




#647064 Cho x, y, z>0 : CMR: $\sum \frac{x^{3}...

Đã gửi bởi royal1534 on 29-07-2016 - 18:51 trong Bất đẳng thức và cực trị

Lời giải:

Ta có bất đẳng thức cần chứng minh tương đương với:

$$\frac{1}{(1+\frac{y}{x})^3}+\frac{1}{(1+\frac{z}{y})^3}+\frac{1}{(1+\frac{x}{z})^3}\geq \frac{3}{8}\;\;\;\;\;\;\;\;\;\;\;\;\;\;\;\;\;\;\;\;\ (1)$$

Đặt $a=\frac{y}{x};b=\frac{z}{y};c=\frac{x}{z}$ thì $abc=1$ và $(1)$ viết lại thành:

$$\frac{1}{(1+a)^3}+\frac{1}{(1+b)^3}+\frac{1}{(1+c)^3}\geq \frac{3}{8}$$

Ta sẽ chứng minh:

$$\frac{1}{(1+a)^3}\geq \frac{3}{8(a^3+\sqrt{a^3}+1)}$$

$$\Leftrightarrow 8a^3+8\sqrt{a^3}+8\geq 3a^3+9a^2+9a+3$$

$$\Leftrightarrow 5a^3+8\sqrt{a^3}+5\geq 9a^2+9a\;\;\;\;\;\;\;\;\;\;\ (2)$$

Bất đẳng thức trên luôn đúng theo $\textrm{AM-GM}:$

  • $$3a^3+6\sqrt{a^3}=3(a^3+\sqrt{a^3}+\sqrt{a^3})\geq 3.3a^2=9a^2$$
  • $$2a^3+2\sqrt{a^3}+5=a^3+a^3+\sqrt{a^3}+\sqrt{a^3}+1+1+1+1+1\geq 9a$$

Cộng $(2)$ vế BĐT trên lại ta được $(2)$ đúng.

Chứng minh tương tự với $b,c$ suy ra:

$$\frac{1}{(1+a)^3}+\frac{1}{(1+b)^3}+\frac{1}{(1+c)^3}\geq \frac{3}{8(a^3+\sqrt{a^3}+1)}+\frac{3}{8(b^3+\sqrt{b^3}+1)}+\frac{3}{8(c^3+\sqrt{c^3}+1)}$$

Cần chứng minh:

$$\frac{1}{a^3+\sqrt{a^3}+1}+\frac{1}{b^3+\sqrt{b^3}+1}+\frac{1}{c^3+\sqrt{c^3}+1}\geq 1$$

Nếu đặt $\sqrt{a^3}=m; \sqrt{b^3}=n; \sqrt{c^3}=p$ thì BĐT này trở nên quen thuộc:

$$\frac{1}{m^2+m+1}+\frac{1}{n^2+n+1}+\frac{1}{p^2+p+1}\geq 1.$$

Chứng minh

Vậy ta có đpcm.

 

P/S

Lời giải của Khánh rất hay. Anh không biết sao mà em tìm được bđt trung gian như vậy

Một lời giải khác:

----------------

Ta cũng đưa bđt về chứng minh:

$\frac{1}{(1+a)^3}+\frac{1}{(1+b)^3}+\frac{1}{(1+c)^3} \geq \frac{3}{8}$

Áp dụng bđt Holder ta có

$(1+a)^3 \leq (1+abc)(1+\frac{a}{b})(1+\frac{a}{c})$

$\Leftrightarrow \frac{1}{(1+a)^3} \geq \frac{bc}{2(a+b)(a+c)}$ (Vì $abc=1$)

Thiết lập các bđt tương tự và cộng lại ta có
$\sum \frac{1}{(1+a)^3} \geq \frac{1}{2}[ \sum \frac{bc}{(a+b)(a+c)}] $

Vậy ta chỉ cần chứng minh 

$\frac{bc}{(a+b)(a+c)}+\frac{ca}{(b+c)(b+a)}+\frac{ab}{(c+b)(c+a)} \geq \frac{3}{4}$

Quy đồng và biến đổi tương đương. BĐT trên tương đương với 

$ab(a+b)+bc(b+c)+ca(a+c) \geq 6abc$

$\leftrightarrow b(a-c)^2+c(a-b)^2+a(b-c)^2 \geq 0$: Đúng

Ta có điều phải chứng minh. Dấu '=' xảy ra khi $a=b=c$




#647005 $(a-b)^2(b-c)^2(c-a)^2$

Đã gửi bởi royal1534 on 29-07-2016 - 12:01 trong Tài liệu, chuyên đề, phương pháp về Bất đẳng thức

Cái đầu tiên là "thành viên VMF" anh ơi :V




#646977 Cmr:$a+b+c+\sqrt{3}\geq 8abc(\frac{1}...

Đã gửi bởi royal1534 on 29-07-2016 - 06:36 trong Bất đẳng thức và cực trị

Cho các số thực dương a,b,c thỏa mãn $ab+bc+ca\leq 1$. Chứng minh rằng:

$a+b+c+\sqrt{3}\geq 8abc(\frac{1}{a^2+1}+\frac{1}{b^2+1}+\frac{1}{c^2+1})$

Áp dụng bất đẳng thức AM-GM ta có
$a^2+1=a^2+\frac{1}{3}+\frac{1}{3}+\frac{1}{3} \geq 4\sqrt[4]{\frac{a^2}{27}}=4\frac{\sqrt{a}}{\sqrt[4]{27}}$
Tương tự:$b^2+1 \geq 4\frac{\sqrt{b}}{\sqrt[4]{27}}$
                $c^2+1 \geq 4\frac{\sqrt{c}}{\sqrt[4]{27}}$
Suy ra $VP \leq 8abc.\sum \frac{\sqrt[4]{27}}{4\sqrt{a}}=2\sqrt[4]{27}(\sum bc\sqrt{a})$
Áp dụng bất đẳng thức AM-GM với nhận xét $1 \geq ab+bc+ca$ ta có
$ab+bc+ca \geq 3\sqrt[3]{a^2b^2c^2} \geq 3\sqrt[3]{a^2b^2c^2}.\sqrt[3]{ab+bc+ca}$
$\Leftrightarrow \sqrt[3]{(ab+bc+ca)^2} \geq 3\sqrt[3]{a^2b^2c^2} $
$\Leftrightarrow (ab+bc+ca)^2 \geq 27a^2b^2c^2 $
$\Leftrightarrow \sqrt{ab+bc+ca} \geq \sqrt[4]{27}\sqrt{abc}$
$\Leftrightarrow \frac{\sqrt{ab+bc+ca}}{\sqrt[4]{27}} \geq \sqrt{abc}$
Suy ra: $\sum bc\sqrt{a} =\sqrt{abc}(\sqrt{bc}+\sqrt{ca}+\sqrt{ab}) \leq \frac{\sqrt{ab+bc+ca}}{\sqrt[4]{27}}.\sqrt{3(ab+bc+ca)}=(ab+bc+ca).\frac{\sqrt{3}}{\sqrt[4]{27}}$
Nên ta có $VP \leq 2\sqrt[4]{27}.(ab+bc+ca).\frac{\sqrt{3}}{\sqrt[4]{27}}=2\sqrt{3}(ab+bc+ca)$
Áp dụng bất đẳng thức AM-GM ta có
$VT=a+b+c+\sqrt{3} \geq 2\sqrt{\sqrt{3}(a+b+c)} \geq 2 \sqrt{3(ab+bc+ca)}$
Vậy ta quy bài toán về chứng minh:
$2\sqrt{3(ab+bc+ca)} \geq 2\sqrt{3}(ab+bc+ca) $
$\leftrightarrow 3(ab+bc+ca) \geq 3(ab+bc+ca)^2$
$\leftrightarrow (ab+bc+ca)(1-ab-bc-ca) \geq 0$: Đúng vì $0<ab+bc+ca \leq 1$
Ta có Đpcm. Đẳng thức xảy ra khi $a=b=c=\frac{1}{\sqrt{3}}$




#646555 $Max P= \frac{1}{2x^2+z}+\frac{1...

Đã gửi bởi royal1534 on 26-07-2016 - 13:29 trong Bất đẳng thức và cực trị

Cho $x,y,z$ là các số thực thõa $1 \leq x,y,z \leq 2$

Tìm GTLN của biểu thức $P= \frac{1}{2x^2+z}+\frac{1}{2y^2+z}-\frac{2xy+1}{xy(z+2)} $




#645762 $P = \sqrt{5x^2+xy+3y^2}+ \sqrt{3x^2+xy+5y^2...

Đã gửi bởi royal1534 on 21-07-2016 - 02:48 trong Bất đẳng thức và cực trị

Phương pháp gì để đoán được lượng đại diện vậy anh

Em sử dụng công thức tổng quát sau:

$ma^2+nb^2+pab=\frac{4mn-p^2}{4(m+n+p)}(a-b)^2+\frac{[a(2m+p)+b(2n+p)]^2}{4(m+n+p)}.$

Từ đó có thể làm trội lên vì $(a-b)^2 \geq 0$  :D

-------------
Bài trên có thể dùng bđt Mincopxki giải tự nhiên hơn  :lol:




#645594 $P = \sqrt{5x^2+xy+3y^2}+ \sqrt{3x^2+xy+5y^2...

Đã gửi bởi royal1534 on 19-07-2016 - 23:02 trong Bất đẳng thức và cực trị

Ta chứng minh được một loạt các bđt sau bằng biến đổi tương đương:

$\sqrt{5x^2+xy+3y^2} \geq \frac{7y+11x}{6} \Leftrightarrow (x-y)^2 \geq 0$

$\sqrt{3x^2+xy+5y^2} \geq \frac{7x+11y}{6} \Leftrightarrow (x-y)^2 \geq 0$

Tương tự : $\sqrt{x^2+xy+2y^2} \geq \frac{3x+5y}{4}$

                   $\sqrt{2x^2+xy+y^2} \geq \frac{5x+3y}{4}$

Cộng các bất đẳng thức vừa tim được ta có

$VT \geq \frac{7x+11y+11x+7y}{6}+\frac{3x+5y+5x+3y}{4}=5(x+y)=5.2016=10080$

Dấu '=' xảy ra khi $x=y=1008$




#645203 $\frac{a+b+c+\sqrt[3]{abc}}{4}...

Đã gửi bởi royal1534 on 16-07-2016 - 20:24 trong Bất đẳng thức và cực trị

Cho $a,b,c$ là các số thực dương. Chứng minh rằng:

$$\frac{a+b+c+\sqrt[3]{abc}}{4}\geq \frac{\sqrt{ab}+\sqrt{bc}+\sqrt{ca}}{3}.$$

Đặt $\sqrt[3]{a}=x,\sqrt[3]{b}=y,\sqrt[3]{c}=z$

Ta cần chứng minh: $3(a^3+b^3+c^3+abc) \geq 4ab\sqrt{ab}+4bc\sqrt{bc}+4ca\sqrt{ca}$

Sử dụng bất đẳng thức AM-GM. Áp dụng liên tục 2 bđt sau:

1.$ a^3+b^3 \geq ab(a+b) \geq 2ab\sqrt{ab} $

Tương tự $\Rightarrow 2(a^3+b^3+c^3) \geq 2ab\sqrt{ab}+2bc\sqrt{bc}+2ca\sqrt{ca}$

2.$ a^3+b^3+c^3+3abc \geq ab(a+b)+bc(b+c)+ca(a+c) \geq 2ab\sqrt{ab}+2bc\sqrt{bc}+2ca\sqrt{ca}$ (BĐT Schur)

Cộng 2 bất đẳng thức vừa tìm được ta có đpcm. Dấu '=' xảy ra khi $x=y=z$ hay $a=b=c$




#645118 Cho các số thực $a,b$ thỏa mãn: $a^2+b^2\le 1$. Chứn...

Đã gửi bởi royal1534 on 16-07-2016 - 03:53 trong Bất đẳng thức và cực trị

Xét $c^2+d^2 \geq 1$. Kết hợp $a^2+b^2 \leq 1$: Ta có $VT \geq 0 \geq VP$ (Đpcm)

Xét $c^2+d^2 \leq 1$.Kết hợp giả thiết $a^2+b^2 \leq 1$:

Áp dụng bđt Cauchy ta có $2=a^2+b^2+c^2+d^2 \geq 2(ac+bd) \leftrightarrow 1 \geq ac+bd$

BĐT cần chứng minh tương đương với

$|ac+bd-1| \geq \sqrt{(1-a^2-b^2)(1-c^2-d^2)}$

$\leftrightarrow 1-ac-bd \geq \sqrt{(1-a^2-b^2)(1-c^2-d^2)}$

Áp dụng bđt AM-GM ta có :

$\sqrt{(1-a^2-b^2)(1-c^2-d^2)} \leq \frac{2-a^2-b^2-c^2-d^2}{2} \leq \frac{2-2ac-2bd}{2}=1-ac-bd$

Ta có điều phải chứng minh. Dấu '=' xảy ra khi $a=b=c=d=\frac{1}{\sqrt{2}}$




#644705 Chứng minh $\sum \frac{a}{\sqrt{(b+c)...

Đã gửi bởi royal1534 on 12-07-2016 - 18:47 trong Bất đẳng thức và cực trị

Gọi P là biểu thức VT và đặt:

$S=a[(b+c)^2+5c^2]+b[(c+a)^2+5a^2]+c[(a+b)^2+5b^2]$.

Sử dụng BĐT Holder ta có: $P^2.S\geq (a+b+c)^3$.

Vậy ta chứng minh:

$(a+b+c)^3\geq S\Leftrightarrow a(a-b)^2+b(b-c)^2+c(c-a)^2\geq 0$.

BĐT cuối luôn đúng.

Dấu bằng xảy ra khi $a=b=c$.

Bạn nhầm rồi thì phải. 

BĐT cần chứng minh là $(a+b+c)^3 \geq \sum a[(b+c)^2+5c^2] \leftrightarrow a^3+b^3+c^3+2(a^2b+b^2c+c^2a) \geq 3(ab^2+bc^2+ca^2)$
BĐT trên không tương đương với bđt cuối cùng của bạn !




#644297 Cho $a,b,c$ là các số thực thỏa mãn: $\sum \frac...

Đã gửi bởi royal1534 on 10-07-2016 - 03:01 trong Bất đẳng thức và cực trị

Phương pháp tiếp tuyến.

Bạn có thể phân tích theo ý bạn nói được không :~ . Cám ơn




#644296 CMR: $(a^2+2)(b^2+2)(c^2+2) \geq 3(a+b+c)^2+(abc-1)^2$

Đã gửi bởi royal1534 on 10-07-2016 - 02:25 trong Bất đẳng thức và cực trị

Cho a,b,c dương. Chứng minh rằng:

$(a^2+2)(b^2+2)(c^2+2) \geq 3(a+b+c)^2+(abc-1)^2$

Sử dụng một bổ đề quen thuộc sau:  
 

Với $a,b,c$ dương ta có: $a^2+b^2+c^2+2abc+1 \geq 2(ab+bc+ca)$

 

------------------------------
Thực hiện phép khai triển. BĐT đã cho tương đương với: 

$a^2+b^2+c^2+2abc+1+2(a^2b^2+b^2c^2+c^2a^2)-6(ab+bc+ca)+6 \geq 0$

 

Áp dụng bổ đề trên. Ta có
 

$VT \geq 2(a^2b^2+b^2c^2+c^2a^2)+2(ab+bc+ca)-6(ab+bc+ca)+6=2(ab-1)^2+2(bc-1)^2+2(ca-1)^2 \geq 0$ 
 

Ta có điều phải chứng minh. Đẳng thức xảy ra khi $a=b=c$